edited by
1,160 views

1 Answer

1 votes
1 votes
$ma[md(a), mn(md(b), a), mn(ab, md(ac))] $ where $a = -2, b = -3, c = 4 $

      $ma[|-2|, mn(|-3|, -2), mn(6, |-8|)] $

      $ma[2, mn(3, -2), mn(6, 8)] $

      $ma[2, -2,6] $

      $Maximum$  $value = 6$
edited by

Related questions

1 votes
1 votes
1 answer
1
makhdoom ghaya asked Oct 28, 2017
799 views
Given that a b, then the relation ma[md(a), mn(a, b)] = mn[a, md(ma(a, b))] does not hold if:a < 0, b < 0 a 0, b 0a 0, b < 0, |a| < |b|a 0, b < 0, |a| |b|
3 votes
3 votes
1 answer
2
makhdoom ghaya asked Oct 28, 2017
1,078 views
A and B walk from X to Y, a distance of 27 km at 5 kmph and 7 kmph respectively. B reaches Y and immediately turns back meeting A at Z. What is the distance from X to Z?2...
2 votes
2 votes
1 answer
4
makhdoom ghaya asked Oct 28, 2017
3,103 views
A man buys spirit at Rs. 60 per litre, adds water to it and then sells it at Rs. 75 per litre. What is the ratio of spirit to water if his profit in the deal is 37.5%?9 :...
2 votes
2 votes
1 answer
5
makhdoom ghaya asked Oct 28, 2017
822 views
What is the smallest number, which when increased by 5 is completely divisible by 8, 11 and 24?264 259269 None of these